Difference between revisions of "1997 PMWC Problems/Problem I1"

(New page: 837 1/420)
 
 
(3 intermediate revisions by 2 users not shown)
Line 1: Line 1:
837 1/420
+
== Problem ==
 +
Evaluate <math>29 \frac{27}{28} \times 27\frac{14}{15}</math>.
 +
 
 +
== Solution 1 ==
 +
<math>\frac{29 \cdot 28 + 27}{28} \cdot \frac{27 \cdot 15 + 14}{15} = \frac{839 \cdot 419}{420} = \frac{839(420 - 1)}{420} = 839 - \frac{839}{420} = 837 \tfrac{1}{420}</math>.
 +
== Solution 2 ==
 +
<math>(30-x)(28-y)=840-28x-30y+xy</math>, so for <math>x=\tfrac1{28},\ y=\tfrac1{15}</math> the result is <math>840-1-2+\tfrac1{420}=837\tfrac1{420}</math>.
 +
 
 +
== See Also ==
 +
{{PMWC box|year=1997|before=First question|num-a=I2}}
 +
 
 +
[[Category:Introductory Algebra Problems]]

Latest revision as of 10:47, 22 May 2014

Problem

Evaluate $29 \frac{27}{28} \times 27\frac{14}{15}$.

Solution 1

$\frac{29 \cdot 28 + 27}{28} \cdot \frac{27 \cdot 15 + 14}{15} = \frac{839 \cdot 419}{420} = \frac{839(420 - 1)}{420} = 839 - \frac{839}{420} = 837 \tfrac{1}{420}$.

Solution 2

$(30-x)(28-y)=840-28x-30y+xy$, so for $x=\tfrac1{28},\ y=\tfrac1{15}$ the result is $840-1-2+\tfrac1{420}=837\tfrac1{420}$.

See Also

1997 PMWC (Problems)
Preceded by
First question
Followed by
Problem I2
I: 1 2 3 4 5 6 7 8 9 10 11 12 13 14 15
T: 1 2 3 4 5 6 7 8 9 10